Table of Contents

Proving that Eigenvalues of Hermitian Matrices are Real

The Prompt

Take the Hermitian adjoint of the following statement:

$$ A\vert v\rangle=\lambda\vert v\rangle $$

Context

This SWBQ

Wrap Up


Personal Tools